LSAT and Law School Admissions Forum

Get expert LSAT preparation and law school admissions advice from PowerScore Test Preparation.

 Administrator
PowerScore Staff
  • PowerScore Staff
  • Posts: 8916
  • Joined: Feb 02, 2011
|
#36274
Complete Question Explanation

Strengthen. The correct answer choice is (A)

The columnist points out that much of Northern America and Western Europe is more heavily
forested, has less acid rain, and has better air quality now than 50 years ago, and grants that the
improvement may be largely due to policies advocated by environmentalists. The columnist then
concludes that the improvement lends support to people who argue that excessive restriction of the
exploitation of resources may make it economically diffi cult to pay for the future protection of the
environment. In layman’s terms, the environment is better, but even though that may be because of
pro-environment policies, if we don’t use our natural resources then we may not have enough money
in the future to continue to apply pro-environment policies.

Remember that in any argument your primary task is to identify the conclusion and supporting
premises, and assess the validity of the argument. If you sensed any holes in the argument (and there
are holes, such as that 50 years over a limited area is not necessarily enough of a sample to prove the
claims, or that the improvement may not have necessarily have come from the policies) remember
that in a Strengthen question you can look for an answer that eliminates such a gap in logic.

Answer choice (A): This is the correct answer choice. If nations did not sustain their wealth by
using their natural resources, then the idea that overly protecting their resources would lead to a
lack of wealth would be false. Thus, this answer supports the claim that restricting the use of natural
resources may diminish the wealth necessary to sustain the pro-environment policies.

Answer choice (B): Technology is extraneous to the issues dealt with in the stimulus, so this choice
is irrelevant and incorrect.

Answer choice (C): Regardless of the cause of the majority of ecological disasters, humans can
still be the cause of this particular disaster. Thus, this answer choice does not affect the issue of
environmental policy.

Answer choice (D): This was the most popular incorrect answer. This answer dwells on what would
have occurred in the past had a different approach been used whereas the argument indicates that
fi nances may dictate a certain course of action in the future. Aside from the fact that what would
have occurred in the past does not mean the same would occur in the future, this answer does not
strengthen the reasoning because it is not relevant to the argument that excessively strict policies may
result in insuffi cient funds to sustain those policies.

Some students see this answer as strengthening the idea that the policies of the environmentalists
have resulted in the environmental improvement, but the author already conceded that point in the
argument so it needed no further support.

Answer choice (E): This choice suggests that a concern for the environment causes an increase in
wealth. If this is the case, then the restrictive environmental policies are benefi cial, and it may be
that they do not diminish the nation’s wealth. Thus, this answer choice weakens the reasoning in the
stimulus.
 lsat2016
  • Posts: 59
  • Joined: May 29, 2016
|
#26160
Hello,

Could anyone explain why A is correct and D is incorrect? I read in another forum that D is considered a premise booster but I couldn't understand why this is the case.

Thank you!
 Clay Cooper
PowerScore Staff
  • PowerScore Staff
  • Posts: 241
  • Joined: Jul 03, 2015
|
#26265
Hi Lsat2016,

Thanks for your question.

Answer choice A is correct because it strengthens our conclusion. The conclusion states that the evidence lends credibility to the claims of some that excessive protection of environmental resources could diminish wealth necessary to employ such policies of protection. If A is true, this possible decrease in wealth becomes more likely, and thus the conclusion that it is a credible possibility is strengthened.

Answer choice D is incorrect because its being true would suggest that a compromise like that for which the author seems to be arguing would have not led to the environmental improvement we actually have seen, and thus that the situation is no less serious than the environmentalists claim - going against what the conclusion states.
 jrc3813
  • Posts: 53
  • Joined: Apr 16, 2017
|
#38751
I have a question about the conclusion/premises of the argument. Is the idea that wealth is neccesary to adopt environmental policies a premise? But the conclusion is that natural resource utilization is neccesary to generate that wealth? So our task is to lend support to the the idea that resource utilization is required to generate that wealth and thus the environmental policies? Or is the idea that wealth is neccesary at all also open to debate?
 AthenaDalton
PowerScore Staff
  • PowerScore Staff
  • Posts: 296
  • Joined: May 02, 2017
|
#38807
Hi jrc,

I think you're on the right track in identifying the parts of the argument.

Premise 1: the environment in North America and Europe has improved over the past 50 years

Premise 2: excessive restrictions on the use of natural resources makes it economically difficult to pay for environmental protection

Conclusion: continuing the restrictions on the use of natural resources may decrease these countries' wealth, thereby limiting their ability to continue applying their pro-environment policies in the future

The issue of whether wealth is necessary to sustain pro-environment policies is at the heart of the conclusion, so answer choices which support that idea will strengthen the argument.

I hope this clarifies things -- best of luck studying!
 Jon@an
  • Posts: 4
  • Joined: Aug 18, 2019
|
#71678
I think I had a harder time with this question because it was hard to see a specific gap in the argument and come up with a prephrase - am I missing something, or is the gap here just that the support (environmental improvement) doesn't do anything for the conclusion (excessively restrictive policies may diminish wealth)?
 Jeremy Press
PowerScore Staff
  • PowerScore Staff
  • Posts: 1000
  • Joined: Jun 12, 2017
|
#71689
Hi Jon@an,

I think you're definitely on the right track with the "gap" in the argument here. The problem with the premises is they don't give us the precise reason why environmental improvement has occurred over the past five decades. The causal chain that led to the current state of affairs is unclear, though the argument's author has admitted that environmentalist policies could be one factor. The author then concludes, without any real causal evidence, that one very specific chain of causation warrants allowance of use of natural resources. The author believes that use of natural resources leads to greater (or sustained) national wealth, which in turn leads to the ability to sustain environmental policies, which themselves bring about environmental improvement. To assist that conclusion, we can look in the answer choices for anything that affirms that causal chain. Answer choice A goes to the first link in the chain: i.e., use of natural resources will indeed lead to greater (or at least sustained) national wealth.

I hope this helps!

Jeremy
User avatar
 CJ12345:
  • Posts: 56
  • Joined: May 25, 2023
|
#103680
Hi, Powerscore,
Does this kind of harder strengthen question have different strategies to approach them? It seems like we cannot use the normal strategies in which we find the conclusion and try to find an AC that makes the conclusion more likely to be valid. It seems like this Q is like an NA question in which we are trying to find the assumption.
That is why I did not choose A since, at first glance, A has nothing to do with the conclusion (which is the improvement of the environment should also be attributed to those people who reject excessive restriction of the use of natural resources). What should I think and approach at this point to reach the correct AC?
 Luke Haqq
PowerScore Staff
  • PowerScore Staff
  • Posts: 747
  • Joined: Apr 26, 2012
|
#103767
Hi CJ12345:!

The administrator summarizes the gist of this argument well: "the environment is better, but even though that may be because of pro-environment policies, if we don’t use our natural resources then we may not have enough money in the future to continue to apply pro-environment policies."

Answer choice (A) connects wealth to use of natural resources. As the administrator notes, "this answer supports the claim that restricting the use of natural resources may diminish the wealth necessary to sustain the pro-environment policies."

Please feel free to explain more why you identified this as an assumption question. The question stem specifically includes the word "strengthen," which should be a clear indicator that one is dealing with a strengthen question. Indeed, I haven't come across any questions that use the word strengthen but are asking for a necessary assumption.

Get the most out of your LSAT Prep Plus subscription.

Analyze and track your performance with our Testing and Analytics Package.